Determining whether $sum_{n=2}^inftyfrac{sin(npi/12)}{ln n}$ converges












0














I'm having issues with determining convergence/divergence of alternating series that use sine and cosine. I'm perfectly clear of how to handle ones with $(-1)^{n+1}$ (and similar) by performing the Absolute Convergence Test and by applying Leibnitz's theorem, but sine and cosine ones are a totally different story.



I simply don't know where to start on this one, for example.



$$sum_{n=2}^inftyfrac{sindfrac{npi}{12}}{ln n}$$










share|cite|improve this question
























  • Welcome to MSE. It is in your best interest that you type your questions (using MathJax) instead of posting links to pictures.
    – José Carlos Santos
    Nov 25 at 15:00










  • Thanks for the welcome! Yeah, I'll take your advice definitely, just need some more time to get to know things.
    – Rose
    Nov 25 at 15:02










  • I've used MathJax to display the series. Feel free to make the rest of the post less wordy, or to clarify whether there's anything you want evaluated besides that one series.
    – J.G.
    Nov 25 at 15:03










  • Thank You! No, that'd be pretty much it I'm interested in right now.
    – Rose
    Nov 25 at 15:06










  • Observe $displaystylesin(npi/12)$ is periodic so its sum cannot be greater than $sum_{n=1}^{12} sin(npi/{12})$.
    – Yadati Kiran
    Nov 25 at 15:15


















0














I'm having issues with determining convergence/divergence of alternating series that use sine and cosine. I'm perfectly clear of how to handle ones with $(-1)^{n+1}$ (and similar) by performing the Absolute Convergence Test and by applying Leibnitz's theorem, but sine and cosine ones are a totally different story.



I simply don't know where to start on this one, for example.



$$sum_{n=2}^inftyfrac{sindfrac{npi}{12}}{ln n}$$










share|cite|improve this question
























  • Welcome to MSE. It is in your best interest that you type your questions (using MathJax) instead of posting links to pictures.
    – José Carlos Santos
    Nov 25 at 15:00










  • Thanks for the welcome! Yeah, I'll take your advice definitely, just need some more time to get to know things.
    – Rose
    Nov 25 at 15:02










  • I've used MathJax to display the series. Feel free to make the rest of the post less wordy, or to clarify whether there's anything you want evaluated besides that one series.
    – J.G.
    Nov 25 at 15:03










  • Thank You! No, that'd be pretty much it I'm interested in right now.
    – Rose
    Nov 25 at 15:06










  • Observe $displaystylesin(npi/12)$ is periodic so its sum cannot be greater than $sum_{n=1}^{12} sin(npi/{12})$.
    – Yadati Kiran
    Nov 25 at 15:15
















0












0








0







I'm having issues with determining convergence/divergence of alternating series that use sine and cosine. I'm perfectly clear of how to handle ones with $(-1)^{n+1}$ (and similar) by performing the Absolute Convergence Test and by applying Leibnitz's theorem, but sine and cosine ones are a totally different story.



I simply don't know where to start on this one, for example.



$$sum_{n=2}^inftyfrac{sindfrac{npi}{12}}{ln n}$$










share|cite|improve this question















I'm having issues with determining convergence/divergence of alternating series that use sine and cosine. I'm perfectly clear of how to handle ones with $(-1)^{n+1}$ (and similar) by performing the Absolute Convergence Test and by applying Leibnitz's theorem, but sine and cosine ones are a totally different story.



I simply don't know where to start on this one, for example.



$$sum_{n=2}^inftyfrac{sindfrac{npi}{12}}{ln n}$$







sequences-and-series convergence






share|cite|improve this question















share|cite|improve this question













share|cite|improve this question




share|cite|improve this question








edited Nov 25 at 15:10









Blue

47.5k870151




47.5k870151










asked Nov 25 at 14:58









Rose

65




65












  • Welcome to MSE. It is in your best interest that you type your questions (using MathJax) instead of posting links to pictures.
    – José Carlos Santos
    Nov 25 at 15:00










  • Thanks for the welcome! Yeah, I'll take your advice definitely, just need some more time to get to know things.
    – Rose
    Nov 25 at 15:02










  • I've used MathJax to display the series. Feel free to make the rest of the post less wordy, or to clarify whether there's anything you want evaluated besides that one series.
    – J.G.
    Nov 25 at 15:03










  • Thank You! No, that'd be pretty much it I'm interested in right now.
    – Rose
    Nov 25 at 15:06










  • Observe $displaystylesin(npi/12)$ is periodic so its sum cannot be greater than $sum_{n=1}^{12} sin(npi/{12})$.
    – Yadati Kiran
    Nov 25 at 15:15




















  • Welcome to MSE. It is in your best interest that you type your questions (using MathJax) instead of posting links to pictures.
    – José Carlos Santos
    Nov 25 at 15:00










  • Thanks for the welcome! Yeah, I'll take your advice definitely, just need some more time to get to know things.
    – Rose
    Nov 25 at 15:02










  • I've used MathJax to display the series. Feel free to make the rest of the post less wordy, or to clarify whether there's anything you want evaluated besides that one series.
    – J.G.
    Nov 25 at 15:03










  • Thank You! No, that'd be pretty much it I'm interested in right now.
    – Rose
    Nov 25 at 15:06










  • Observe $displaystylesin(npi/12)$ is periodic so its sum cannot be greater than $sum_{n=1}^{12} sin(npi/{12})$.
    – Yadati Kiran
    Nov 25 at 15:15


















Welcome to MSE. It is in your best interest that you type your questions (using MathJax) instead of posting links to pictures.
– José Carlos Santos
Nov 25 at 15:00




Welcome to MSE. It is in your best interest that you type your questions (using MathJax) instead of posting links to pictures.
– José Carlos Santos
Nov 25 at 15:00












Thanks for the welcome! Yeah, I'll take your advice definitely, just need some more time to get to know things.
– Rose
Nov 25 at 15:02




Thanks for the welcome! Yeah, I'll take your advice definitely, just need some more time to get to know things.
– Rose
Nov 25 at 15:02












I've used MathJax to display the series. Feel free to make the rest of the post less wordy, or to clarify whether there's anything you want evaluated besides that one series.
– J.G.
Nov 25 at 15:03




I've used MathJax to display the series. Feel free to make the rest of the post less wordy, or to clarify whether there's anything you want evaluated besides that one series.
– J.G.
Nov 25 at 15:03












Thank You! No, that'd be pretty much it I'm interested in right now.
– Rose
Nov 25 at 15:06




Thank You! No, that'd be pretty much it I'm interested in right now.
– Rose
Nov 25 at 15:06












Observe $displaystylesin(npi/12)$ is periodic so its sum cannot be greater than $sum_{n=1}^{12} sin(npi/{12})$.
– Yadati Kiran
Nov 25 at 15:15






Observe $displaystylesin(npi/12)$ is periodic so its sum cannot be greater than $sum_{n=1}^{12} sin(npi/{12})$.
– Yadati Kiran
Nov 25 at 15:15












1 Answer
1






active

oldest

votes


















1














We use the Dirichlet's test.



For that, we observe that $frac{1}{log (n)}$ is decreasing and tending to $0$ as $nto infty.$ Next we have to show that
$$S_M=sum_{n=1}^{M}sin(npi/12)$$ is bounded.



For this use the following equation:



$$sum_{k=1}^{n}sin kx=frac{cosleft(frac{1}2xright)-cosleft(n+frac{1}2right)x}{2sin(x/2)},xneq0,pmpi,pm 2pi,...$$






share|cite|improve this answer





















  • Ah, Dirichlet's theorem, I'd never think of it, thanks! Could you just explain the way you prove that the partial sums are bounded?
    – Rose
    Nov 25 at 15:41











Your Answer





StackExchange.ifUsing("editor", function () {
return StackExchange.using("mathjaxEditing", function () {
StackExchange.MarkdownEditor.creationCallbacks.add(function (editor, postfix) {
StackExchange.mathjaxEditing.prepareWmdForMathJax(editor, postfix, [["$", "$"], ["\\(","\\)"]]);
});
});
}, "mathjax-editing");

StackExchange.ready(function() {
var channelOptions = {
tags: "".split(" "),
id: "69"
};
initTagRenderer("".split(" "), "".split(" "), channelOptions);

StackExchange.using("externalEditor", function() {
// Have to fire editor after snippets, if snippets enabled
if (StackExchange.settings.snippets.snippetsEnabled) {
StackExchange.using("snippets", function() {
createEditor();
});
}
else {
createEditor();
}
});

function createEditor() {
StackExchange.prepareEditor({
heartbeatType: 'answer',
autoActivateHeartbeat: false,
convertImagesToLinks: true,
noModals: true,
showLowRepImageUploadWarning: true,
reputationToPostImages: 10,
bindNavPrevention: true,
postfix: "",
imageUploader: {
brandingHtml: "Powered by u003ca class="icon-imgur-white" href="https://imgur.com/"u003eu003c/au003e",
contentPolicyHtml: "User contributions licensed under u003ca href="https://creativecommons.org/licenses/by-sa/3.0/"u003ecc by-sa 3.0 with attribution requiredu003c/au003e u003ca href="https://stackoverflow.com/legal/content-policy"u003e(content policy)u003c/au003e",
allowUrls: true
},
noCode: true, onDemand: true,
discardSelector: ".discard-answer"
,immediatelyShowMarkdownHelp:true
});


}
});














draft saved

draft discarded


















StackExchange.ready(
function () {
StackExchange.openid.initPostLogin('.new-post-login', 'https%3a%2f%2fmath.stackexchange.com%2fquestions%2f3012942%2fdetermining-whether-sum-n-2-infty-frac-sinn-pi-12-ln-n-converges%23new-answer', 'question_page');
}
);

Post as a guest















Required, but never shown

























1 Answer
1






active

oldest

votes








1 Answer
1






active

oldest

votes









active

oldest

votes






active

oldest

votes









1














We use the Dirichlet's test.



For that, we observe that $frac{1}{log (n)}$ is decreasing and tending to $0$ as $nto infty.$ Next we have to show that
$$S_M=sum_{n=1}^{M}sin(npi/12)$$ is bounded.



For this use the following equation:



$$sum_{k=1}^{n}sin kx=frac{cosleft(frac{1}2xright)-cosleft(n+frac{1}2right)x}{2sin(x/2)},xneq0,pmpi,pm 2pi,...$$






share|cite|improve this answer





















  • Ah, Dirichlet's theorem, I'd never think of it, thanks! Could you just explain the way you prove that the partial sums are bounded?
    – Rose
    Nov 25 at 15:41
















1














We use the Dirichlet's test.



For that, we observe that $frac{1}{log (n)}$ is decreasing and tending to $0$ as $nto infty.$ Next we have to show that
$$S_M=sum_{n=1}^{M}sin(npi/12)$$ is bounded.



For this use the following equation:



$$sum_{k=1}^{n}sin kx=frac{cosleft(frac{1}2xright)-cosleft(n+frac{1}2right)x}{2sin(x/2)},xneq0,pmpi,pm 2pi,...$$






share|cite|improve this answer





















  • Ah, Dirichlet's theorem, I'd never think of it, thanks! Could you just explain the way you prove that the partial sums are bounded?
    – Rose
    Nov 25 at 15:41














1












1








1






We use the Dirichlet's test.



For that, we observe that $frac{1}{log (n)}$ is decreasing and tending to $0$ as $nto infty.$ Next we have to show that
$$S_M=sum_{n=1}^{M}sin(npi/12)$$ is bounded.



For this use the following equation:



$$sum_{k=1}^{n}sin kx=frac{cosleft(frac{1}2xright)-cosleft(n+frac{1}2right)x}{2sin(x/2)},xneq0,pmpi,pm 2pi,...$$






share|cite|improve this answer












We use the Dirichlet's test.



For that, we observe that $frac{1}{log (n)}$ is decreasing and tending to $0$ as $nto infty.$ Next we have to show that
$$S_M=sum_{n=1}^{M}sin(npi/12)$$ is bounded.



For this use the following equation:



$$sum_{k=1}^{n}sin kx=frac{cosleft(frac{1}2xright)-cosleft(n+frac{1}2right)x}{2sin(x/2)},xneq0,pmpi,pm 2pi,...$$







share|cite|improve this answer












share|cite|improve this answer



share|cite|improve this answer










answered Nov 25 at 15:09









Hello_World

3,83721630




3,83721630












  • Ah, Dirichlet's theorem, I'd never think of it, thanks! Could you just explain the way you prove that the partial sums are bounded?
    – Rose
    Nov 25 at 15:41


















  • Ah, Dirichlet's theorem, I'd never think of it, thanks! Could you just explain the way you prove that the partial sums are bounded?
    – Rose
    Nov 25 at 15:41
















Ah, Dirichlet's theorem, I'd never think of it, thanks! Could you just explain the way you prove that the partial sums are bounded?
– Rose
Nov 25 at 15:41




Ah, Dirichlet's theorem, I'd never think of it, thanks! Could you just explain the way you prove that the partial sums are bounded?
– Rose
Nov 25 at 15:41


















draft saved

draft discarded




















































Thanks for contributing an answer to Mathematics Stack Exchange!


  • Please be sure to answer the question. Provide details and share your research!

But avoid



  • Asking for help, clarification, or responding to other answers.

  • Making statements based on opinion; back them up with references or personal experience.


Use MathJax to format equations. MathJax reference.


To learn more, see our tips on writing great answers.





Some of your past answers have not been well-received, and you're in danger of being blocked from answering.


Please pay close attention to the following guidance:


  • Please be sure to answer the question. Provide details and share your research!

But avoid



  • Asking for help, clarification, or responding to other answers.

  • Making statements based on opinion; back them up with references or personal experience.


To learn more, see our tips on writing great answers.




draft saved


draft discarded














StackExchange.ready(
function () {
StackExchange.openid.initPostLogin('.new-post-login', 'https%3a%2f%2fmath.stackexchange.com%2fquestions%2f3012942%2fdetermining-whether-sum-n-2-infty-frac-sinn-pi-12-ln-n-converges%23new-answer', 'question_page');
}
);

Post as a guest















Required, but never shown





















































Required, but never shown














Required, but never shown












Required, but never shown







Required, but never shown

































Required, but never shown














Required, but never shown












Required, but never shown







Required, but never shown







Popular posts from this blog

Quarter-circle Tiles

build a pushdown automaton that recognizes the reverse language of a given pushdown automaton?

Mont Emei